Solved papers for RAJASTHAN ­ PET Rajasthan PET Solved Paper-2009

done Rajasthan PET Solved Paper-2009 Total Questions - 120

  • question_answer1) Which of the following equation represents a progressive wave?

    A)
     \[y=\,A\cos \,ax\,\sin bt\]

    B)
     \[y=\,A\,\sin \,bt\]

    C)
     \[y=\,A\,\cos (ax+bt)\]  

    D)
     \[y=\,A\,\tan (ax+bt)\]

    View Answer play_arrow
  • question_answer2) The frequency and velocity of sound wave are 600 Hz and 360 m/s respectively. Phase difference between two particles of medium are\[60{}^\circ ,\]the minimum distance between these two particles will be

    A)
     10 cm           

    B)
     15 cm

    C)
     20 cm           

    D)
     50 cm

    View Answer play_arrow
  • question_answer3) The heat required to increase the temperature of 4 moles of a monoatomic ideal gas from 273 K to 473 K at constant volume is

    A)
     200 H            

    B)
     400 R

    C)
     800 J           

    D)
     1200 R

    View Answer play_arrow
  • question_answer4) A solid sphere rolls without slipping on the roof. The ratio of its rotational kinetic energy and its total kinetic energy is

    A)
     2/5

    B)
     4/5

    C)
     2/7    

    D)
     3/7

    View Answer play_arrow
  • question_answer5) A black body at a temperature of 2600 K has the wavelength corresponding to maximum emission 1200\[\overset{o}{\mathop{\text{A}}}\,\]. Assuming the moon to be perfectly black body the temperature of the moon, if the wavelength corresponding to maximum emission is 5000\[\overset{o}{\mathop{\text{A}}}\,\] is

    A)
     7800 K          

    B)
     6240 K

    C)
     5240 K          

    D)
     3640 K

    View Answer play_arrow
  • question_answer6) When the light enters from air to glass, for which colour the angle of deviation is maximum?

    A)
     Red              

    B)
     Yellow

    C)
     Blue              

    D)
     Violet

    View Answer play_arrow
  • question_answer7) Two waves are represent by \[{{y}_{1}}=A\sin (kx-\omega t)\,and\,{{y}_{2}}=A\cos \,(kx-\omega t)\] The amplitude of resultant wave is

    A)
     4A               

    B)
     2A

    C)
     \[\sqrt{2}A\]             

    D)
     A

    View Answer play_arrow
  • question_answer8) Which of the following event, support the quantum nature of light?

    A)
     Diffraction

    B)
     Polarization

    C)
     Interference

    D)
     Photoelectric effect

    View Answer play_arrow
  • question_answer9) The linear momentum of photon is p. The wavelength of photon is \[\lambda \], then (\[h\]is Planck?s constant)

    A)
     \[\lambda =hp\]

    B)
     \[\lambda =\frac{h}{p}\]

    C)
     \[\lambda =\frac{p}{h}\]

    D)
     \[\lambda =\frac{{{p}^{2}}}{h}\]

    View Answer play_arrow
  • question_answer10) When two coherent monochromatic beams of intensity I and 9l interfere, the possible maximum and minimum intensities of the resulting beam are

    A)
     \[9I\] and\[I\]         

    B)
     \[9I\]and\[4I\]

    C)
     \[16I\]and\[4I\]       

    D)
     \[16I\]and\[I\]

    View Answer play_arrow
  • question_answer11) An atom of mass M which is in the state of rest emits a photon of wavelength \[\lambda \] . As a result, the atom will deflect with the kinetic energy equal to (A is Planck's constant)

    A)
     \[\frac{{{h}^{2}}}{M{{\lambda }^{2}}}\]

    B)
     \[\frac{1}{2}\frac{{{h}^{2}}}{M{{\lambda }^{2}}}\]

    C)
     \[\frac{h}{M\lambda }\]

    D)
     \[\frac{1}{2}\frac{h}{M\lambda }\]   

    View Answer play_arrow
  • question_answer12) Two long straight wires are set parallel to each other Each carries a current i in the opposite direction and the separation between them is 2R. The intensity of the magnetic field midway between them is

    A)
     zero              

    B)
     \[\frac{{{\mu }_{0}}i}{4\pi R}\]

    C)
     \[\frac{{{\mu }_{0}}i}{2\pi R}\]             

    D)
     \[\frac{{{\mu }_{0}}i}{\pi R}\]

    View Answer play_arrow
  • question_answer13) Charge Q is placed at the diagonal faced comers of a square and charge q is placed at another two corners of square. The condition for net electric force on Q to be zero will be

    A)
     \[Q=(-2\sqrt{2)}\,q\]

    B)
     \[Q=-\frac{q}{2}\]

    C)
     \[Q=(2\sqrt{2)}\,q\]       

    D)
     \[Q=-\frac{q}{2}\]

    View Answer play_arrow
  • question_answer14) The electric potential at centre of metallic conducting sphere is

    A)
     zero

    B)
     half from potential at surface of sphere

    C)
     equal from potential at surface of sphere

    D)
     twice from potential at surface of sphere

    View Answer play_arrow
  • question_answer15) A capacitor of capacity 10 \[\mu \]F is charged to a potential of 400 Volt. When its both plates are connected by a conducting wire, then heat generated will be

    A)
     \[80\text{ }J\]             

    B)
     \[0.8\text{ }J\]

    C)
     \[8\times {{10}^{-3}}\]         

    D)
      \[8\times {{10}^{-6}}J\]

    View Answer play_arrow
  • question_answer16) Magnet moment of bar magnet is M. The work done to turn the magnet by\[90{}^\circ \]of magnet in direction of magnetic field B will be

    A)
     zero             

    B)
    \[\frac{1}{2}\]MB

    C)
     2 MB          

    D)
     MB

    View Answer play_arrow
  • question_answer17) Voltage V and current\[i\]in AC circuit are given by \[\begin{align}   & V=50\sin (50\,t)\,volt \\  & i=50\,\sin \,\left( 50t+\frac{\pi }{3} \right)mA \\ \end{align}\] The power dissipated in circuit is

    A)
     5.0 W            

    B)
     2.5 W

    C)
     1.25 W           

    D)
     zero

    View Answer play_arrow
  • question_answer18) The maximum voltage in DC circuit is 282 Volt. The effective voltage in AC circuit will be

    A)
     200 V            

    B)
     300 V

    C)
     400 V            

    D)
     564 V

    View Answer play_arrow
  • question_answer19) If L, C and R denote inductance, capacitance and resistance respectively, then which of the following combination has the dimension of time?

    A)
     \[\frac{C}{L}\]

    B)
     \[\frac{1}{RC}\]

    C)
     \[\frac{L}{R}\]

    D)
     \[\frac{RL}{C}\]

    View Answer play_arrow
  • question_answer20) The ratio of momenta of an electron and photon which are accelerated from rest by a potential difference 50 Volt is

    A)
     \[\frac{{{m}_{e}}}{{{m}_{p}}}\]

    B)
     \[\sqrt{\frac{{{m}_{e}}}{{{m}_{p}}}}\]

    C)
     \[\frac{{{m}_{p}}}{{{m}_{e}}}\]

    D)
     \[\sqrt{\frac{{{m}_{p}}}{{{m}_{e}}}}\]

    View Answer play_arrow
  • question_answer21) A wire of resistance\[18\,\Omega \]is divided into three equal parts. These parts are connected in side of triangle, the equivalent resistance of any two corners of triangle will be

    A)
     18\[\Omega \]          

    B)
     9\[\Omega \]

    C)
     6\[\Omega \]             

    D)
     4\[\Omega \]

    View Answer play_arrow
  • question_answer22) Two capacitors of capacity 6 \[\mu \]F and 12 \[\mu \]V in series are connected by potential of 150 V. The potential of capacitor of capacity 12 \[\mu \]F will be

    A)
     25 V             

    B)
     50 V

    C)
     100 V            

    D)
     150 V

    View Answer play_arrow
  • question_answer23) A charged particle enters in a magnetic field whose direction is parallel to velocity of the particle, then the speed of this particle

    A)
     in straight line    

    B)
     in coiled path

    C)
     in circular path    

    D)
     in ellipse path

    View Answer play_arrow
  • question_answer24) Magnetic intensity for an axial point due to a short bar magnet of magnetic moment M is given by

    A)
     \[\frac{{{\mu }_{0}}}{4\pi }\frac{m}{{{d}^{3}}}\]

    B)
     \[\frac{{{\mu }_{0}}M}{4\pi {{d}^{2}}}\]

    C)
     \[\frac{{{\mu }_{0}}}{2\pi }\frac{\mu }{{{d}^{3}}}\]

    D)
     \[\frac{{{\mu }_{0}}M}{2\pi {{d}^{2}}}\]

    View Answer play_arrow
  • question_answer25) Which of the following transition gives the photon of minimum frequency?

    A)
     n = 2 to n = 1      

    B)
     n = 3 to n = 1

    C)
     n = 3 to n = 2     

    D)
     n = 4 to n = 3

    View Answer play_arrow
  • question_answer26)  lonization energy of He+ ion at minimum position is

    A)
     13.6 eV          

    B)
     27.2 eV

    C)
     54.4 eV          

    D)
     68.0 eV

    View Answer play_arrow
  • question_answer27) The half-life period of a radioactive substance is 3 days. Three fourth of substance decays in

    A)
     3 days            

    B)
     6 days

    C)
     9 days            

    D)
     12 days

    View Answer play_arrow
  • question_answer28) If the decay constant of a radioactive substance is \[\lambda \], then its half-life is

    A)
     \[\frac{1}{\lambda }{{\log }_{e}}2\]

    B)
     \[\frac{1}{\lambda }\]

    C)
     \[\lambda {{\log }_{e}}2\]

    D)
     \[\frac{\lambda }{{{\log }_{e}}2}\]

    View Answer play_arrow
  • question_answer29) A galvanometer can be converted into a voltmeter by connecting

    A)
     low resistance in parallel

    B)
     low resistance in series

    C)
     high resistance in parallel

    D)
     high resistance in series

    View Answer play_arrow
  • question_answer30) A heater coil cut into two equal parts and one part is connected with heater. Now heat generated in heater will be

    A)
     twice            

    B)
     half

    C)
     one- fourth        

    D)
     four times

    View Answer play_arrow
  • question_answer31) A particle moves along with x-axis. The position\[x\]of particle with respect to time t from origin given by \[x={{b}_{0}}+{{b}_{1}}t+{{b}_{2}}{{t}_{2}}\]. The acceleration of particle is

    A)
     \[{{b}_{0}}\]                

    B)
     \[{{b}_{1}}\]

    C)
     \[{{b}_{2}}\]                

    D)
     \[2{{b}_{2}}\]

    View Answer play_arrow
  • question_answer32) One end of string of length\[l\]is connected to a particle of mass m and the other to a small peg on a smooth horizontal table. If the particle moves in a circle with speed v, the net force on the particle (directed towards the centre) is

    A)
     zero              

    B)
     \[T-\frac{m{{v}^{2}}}{I}\]

    C)
     \[T\]              

    D)
     \[T+\frac{m{{v}^{2}}}{I}\]

    View Answer play_arrow
  • question_answer33) If the kinetic energy of a body is increased 2 times, its momentum will

    A)
     half

    B)
     remain unchanged

    C)
     be doubled

    D)
     increase \[\sqrt{2}\]times

    View Answer play_arrow
  • question_answer34) The moment of inertia of a solid sphere of mass M and radius R about the tangent on its surface is

    A)
     \[\frac{7}{5}M{{R}^{2}}\]

    B)
     \[\frac{4}{5}M{{R}^{2}}\]

    C)
     \[\frac{2}{5}M{{R}^{2}}\]           

    D)
     \[\frac{1}{2}M{{R}^{2}}\]

    View Answer play_arrow
  • question_answer35) Satellites A and B are revolving around the orbit of earth. The mass of A is 10 times to mass of B. The ratio of time period \[\left( \frac{{{T}_{A}}}{{{T}_{B}}} \right)\] is

    A)
     \[10\]

    B)
     \[1\]

    C)
     \[\frac{1}{5}\]

    D)
     \[\frac{1}{10}\]

    View Answer play_arrow
  • question_answer36) A particle of amplitude A is executing simple harmonic motion. When the potential energy of particle is half of its maximum potential energy,   then   displacement   from   its equilibrium position is

    A)
     \[\frac{A}{4}\]                  

    B)
     \[\frac{A}{3}\]

    C)
     \[\frac{A}{2}\]                  

    D)
     \[\frac{A}{\sqrt{2}}\]

    View Answer play_arrow
  • question_answer37) A simple wave motion represents by\[y=5(\sin 4\pi t+\sqrt{3}\cos 4\pi t)\]. Its amplitude is

    A)
     5                   

    B)
     5\[\sqrt{3}\]

    C)
     \[10\sqrt{3}\]               

    D)
     10

    View Answer play_arrow
  • question_answer38) A liquid does not wet the sides of a solid, if the angle of contact is

    A)
     obtuse           

    B)
     \[90{}^\circ \]

    C)
     acute             

    D)
     zero

    View Answer play_arrow
  • question_answer39) Root means square speed of the molecules of ideal gas is\[v\]. If pressure is increased two times at constant temperature, then the rms speed will become

    A)
     \[\frac{v}{2}\]              

    B)
     \[v\]

    C)
     \[2v\]                 

    D)
     \[4v\]

    View Answer play_arrow
  • question_answer40) 1 mole of gas occupies a volume of 200 ml at 100 mm pressure. What is the volume occupied by two moles of gas at 400 mm pressure and at same temperature?

    A)
     50 mL           

    B)
     100 mL

    C)
     200 mL          

    D)
     400 mL

    View Answer play_arrow
  • question_answer41) The\[s{{p}^{3}}\]hybridisation is present on the central atom of

    A)
     \[HCHO\]          

    B)
     \[BC{{l}_{3}}\]

    C)
     \[PC{{l}_{3}}\]            

    D)
     \[S{{O}_{3}}\]

    View Answer play_arrow
  • question_answer42) The number of unit cells in the 5.85 g crystals of\[NaCl\]are

    A)
     \[1.5\times {{10}^{23}}\]        

    B)
     \[1.5\times {{10}^{22}}\]

    C)
     \[3.0\times {{10}^{22}}\]        

    D)
     \[3.0\times {{10}^{23}}\]

    View Answer play_arrow
  • question_answer43) The oxidation number of sulphur is -1 in

    A)
     \[{{H}_{2}}S\]              

    B)
     \[Fe{{S}_{2}}\]

    C)
     \[C{{S}_{2}}\]               

    D)
     \[C{{u}_{2}}S\]

    View Answer play_arrow
  • question_answer44) For which of the following metals, the electronic configuration of valence shell is not \[{{d}^{6}}\]?

    A)
     Fe (II)              

    B)
    \[Mn\](II)

    C)
     Co (III)             

    D)
     Ni (IV)

    View Answer play_arrow
  • question_answer45) 68 g sugar\[({{C}_{12}}{{H}_{22}}{{O}_{11}})\]is dissolved in 1 kg of water. What is the mole fraction of sugar?

    A)
     0.018            

    B)
     0.036

    C)
     0.0018           

    D)
     0.0036

    View Answer play_arrow
  • question_answer46) The pH of 0.1 M aqueous ammonia \[({{K}_{b}}=1.8\times {{10}^{-5}})\]is.

    A)
     9.13               

    B)
     10.13

    C)
     11.13             

    D)
     12.13

    View Answer play_arrow
  • question_answer47) From the following mixtures, which is not a buffer (concentration level 0.5 M)?

    A)
     \[C{{H}_{3}}COOH+NaOH(2:1)\]

    B)
     \[HCl+N{{H}_{3}}(aq)(1:2)\]

    C)
     \[C{{H}_{3}}COOH+NaOH(1:2)\]

    D)
     \[HCl+N{{H}_{3}}(2:3)\]

    View Answer play_arrow
  • question_answer48) How much volume (in litre) of\[3M\,NaOH\]is obtained from 80 g\[NaOH\]? (Atomic mass of \[Na=23u\])

    A)
     2.67               

    B)
     1.34

    C)
     0.67               

    D)
     0.33

    View Answer play_arrow
  • question_answer49) The initial concentration of sugar solution is 0.12 M. On    doing   fermentation   the concentration of sugar decreases to 0.06 M in 10 h and to 0.045 M in 15 h. The order of the reaction is

    A)
     0.5                  

    B)
     1.0

    C)
     1.5                  

    D)
     2.0

    View Answer play_arrow
  • question_answer50) For the decomposition of\[{{N}_{2}}O\]and\[{{O}_{2}}\]in presence of at, the velocity constant, k is \[k=5\times {{10}^{11}}{{e}^{-30,000/T}}\] For this, the activation energy is (in kJ\[mo{{l}^{-1}}\])

    A)
     2.494             

    B)
     24.94

    C)
     249.4             

    D)
     2494

    View Answer play_arrow
  • question_answer51) The following equilibrium establishes on heating 0.2 mole of\[{{H}_{2}}\]and 1.0 mole of sulphur in 1 L vessel at\[90{}^\circ C\]. \[{{H}_{2}}(g)+S(s){{H}_{2}}S(g);\]  \[K=6.8\times {{10}^{-2}}\] The partial pressure of\[{{H}_{2}}S\]in equilibrium state is

    A)
     4.20               

    B)
     0.42

    C)
     0.21               

    D)
     0.042

    View Answer play_arrow
  • question_answer52) An aqueous solution boils at\[100.2{}^\circ C\]. At which temperature this will freeze. \[({{K}_{b}}=0.5{}^\circ C/m,{{K}_{f}}=1.9{}^\circ C/m)\]

    A)
     \[+0.76\]           

    B)
     \[-0.76\]

    C)
     \[-0.38\]            

    D)
     \[+0.38\]

    View Answer play_arrow
  • question_answer53) The lowest\[p{{K}_{a}}\]value is for

    A)
     phenol            

    B)
    \[m-\]cresol

    C)
     o-cresol           

    D)
     p-cresol

    View Answer play_arrow
  • question_answer54) At room temperature, the least stable compound is

    A)
     \[C{{H}_{3}}COCl\]        

    B)
     \[HCOCl\]

    C)
     \[C{{H}_{3}}COOH\]       

    D)
     \[{{(C{{H}_{3}}CO)}_{2}}O\]

    View Answer play_arrow
  • question_answer55) For the conversion of\[C{{H}_{2}}=C{{H}_{2}}\]into\[HOOC\]. \[C{{H}_{2}}C{{H}_{2}}COOH,\]the minimum number of steps required are

    A)
     2              

    B)
     3

    C)
     4              

    D)
     5

    View Answer play_arrow
  • question_answer56) Which of the following compounds does not give two isomer compounds on reaction with\[N{{H}_{2}}OH\]?

    A)
     \[C{{H}_{3}}CO{{C}_{2}}{{H}_{5}}\]     

    B)
     \[C{{H}_{3}}COC{{H}_{3}}\]

    C)
     \[C{{H}_{3}}CHO\]        

    D)
     \[PhCOC{{H}_{3}}\]

    View Answer play_arrow
  • question_answer57) From the following which is not a reactant, reagent or product in Hoffmann reaction?

    A)
     \[RCON{{H}_{2}}\]

    B)
     \[RN{{H}_{2}}\]

    C)
     \[B{{r}_{2}},O{{H}^{-}}\]

    D)
     \[{{H}_{2}}S{{O}_{4}}\]

    View Answer play_arrow
  • question_answer58) The total number of isomers for cyclic alcohol \[{{C}_{4}}{{H}_{7}}OH\]is

    A)
     2    

    B)
     3     

    C)
     4    

    D)
     5

    View Answer play_arrow
  • question_answer59) The least stable free radical is

    A)
     \[\overset{\bullet }{\mathop{C}}\,{{H}_{2}}C{{H}_{2}}CH{{(C{{H}_{3}})}_{2}}\]

    B)
     \[C{{H}_{3}}\underset{\bullet }{\mathop{C}}\,HCH{{(C{{H}_{3}})}_{2}}\]

    C)
     \[C{{H}_{3}}C{{H}_{2}}\underset{\bullet }{\mathop{C}}\,{{(C{{H}_{3}})}_{2}}\]

    D)
     \[\overset{\bullet }{\mathop{C}}\,{{H}_{3}}\]

    View Answer play_arrow
  • question_answer60) The suitable reagent for the reduction of \[{{C}_{2}}{{H}_{5}}COOH\]into\[{{C}_{3}}{{H}_{7}}OH\]is

    A)
     \[B{{H}_{3}}/THF\]and\[{{H}_{3}}{{O}^{+}}\]

    B)
     \[NaB{{H}_{4}}\]

    C)
     \[Na/EtOH\]

    D)
     \[{{H}_{2}}/\]catalyst

    View Answer play_arrow
  • question_answer61) The blue colour of acidic solution of\[C{{r}_{2}}O_{7}^{2-}\]is not changed into green by

    A)
     \[{{C}_{6}}{{H}_{5}}C{{H}_{2}}OH\]

    B)
     \[{{(C{{H}_{3}})}_{2}}CHOH\]

    C)
     \[C{{H}_{3}}C{{H}_{2}}C{{H}_{2}}C{{H}_{2}}OH\]

    D)
     \[{{(C{{H}_{3}})}_{3}}COH\]

    View Answer play_arrow
  • question_answer62) The reaction of\[{{C}_{2}}{{H}_{5}}Cl\]with\[Li\]and\[CuI\]gives mainly

    A)
     2-butene         

    B)
     1, 3-butadiene

    C)
     n-butane         

    D)
     n-butyl chloride

    View Answer play_arrow
  • question_answer63) The reagent which does not convert n-butyl chloride into zi-butane is

    A)
     \[Zn,\text{ }HCl\]

    B)
     \[LIAl{{H}_{4}}\]

    C)
    \[Mg,\]Anhydrous ether,\[{{H}_{2}}O\]

    D)
     \[{{B}_{2}}{{H}_{6}}\]in THF

    View Answer play_arrow
  • question_answer64) The correct order of stability is

    A)
     Pentane < iso-pentane < neo-pentane

    B)
     iso-pentane < neo-pentane < pentane

    C)
     neo-pentane < iso-pentane < pentane

    D)
     Pentane < iso-pentane < iso-pentane

    View Answer play_arrow
  • question_answer65) The correct order of boiling point of ethyl dimethyl amine , n-butyl amine  and diethyl amine  is

    A)
     \[B>C>A\]          

    B)
     \[B>A>C\]

    C)
     \[A>B>C\]           

    D)
     \[C>B>A\]

    View Answer play_arrow
  • question_answer66) Which of the following compounds does not give iodoform test?

    A)
     \[C{{H}_{3}}COC{{H}_{2}}COO{{C}_{2}}{{H}_{5}}\]

    B)
     \[PhC{{H}_{2}}COC{{H}_{3}}\]

    C)
     \[M{{e}_{3}}C.COC{{H}_{3}}\]

    D)
     \[C{{H}_{3}}COC{{H}_{3}}\]

    View Answer play_arrow
  • question_answer67) The species which acts as both nucleophile and electrophile is

    A)
     \[C{{H}_{3}}CN\]

    B)
     \[N{{H}_{3}}\]

    C)
     \[P{{(C{{H}_{3}})}_{2}}\]

    D)
     \[{{H}_{2}}\]

    View Answer play_arrow
  • question_answer68) The most basic from the following is

    A)
     \[N{{H}_{3}}\]

    B)
     \[C{{H}_{3}}N{{H}_{2}}\]

    C)
     \[N{{F}_{3}}\]

    D)
     \[N{{(Si{{H}_{3}})}_{3}}\]

    View Answer play_arrow
  • question_answer69) The main product of the reaction of benzene with lithium in liquid ammonia and\[EtOH\]is

    A)
     

    B)
     

    C)
     

    D)
     

    View Answer play_arrow
  • question_answer70) The rate of free radical chlorination of\[C{{H}_{4}}\]is

    A)
     equal to\[C{{D}_{4}}\]

    B)
     double the rate of \[C{{D}_{4}}\]

    C)
     12 times the rate of\[C{{D}_{4}}\]

    D)
     less than the rate of\[C{{D}_{4}}\]

    View Answer play_arrow
  • question_answer71) The first ionisation energy difference is maximum for which pair?

    A)
     \[Na,Mg\]          

    B)
     \[K,Ca\]

    C)
     \[Rb,Sr\]           

    D)
     \[Cs,Ba\]

    View Answer play_arrow
  • question_answer72) The ratio of third Bohr orbit radius and second Bohr orbit radius for the hydrogen atom is

    A)
     0.5                 

    B)
     1.5

    C)
     0.75               

    D)
     2.25

    View Answer play_arrow
  • question_answer73) The principal quantum number of Mn for those valence shell orbits in which electrons are filled

    A)
     4, 3              

    B)
     4, 4

    C)
     3, 3              

    D)
     5, 4

    View Answer play_arrow
  • question_answer74) From the generally known oxidation states, the oxidation number is maximum for

    A)
     \[Mn\]             

    B)
     \[Cu\]

    C)
     \[Sn\]              

    D)
     \[Sc\]

    View Answer play_arrow
  • question_answer75) The number of molecules in 180 g of heavy water are

    A)
     \[6.02\times {{10}^{24}}\]       

    B)
     \[6.02\times {{10}^{22}}\]

    C)
     \[5.42\times {{10}^{24}}\]       

    D)
     \[5.42\times {{10}^{23}}\]

    View Answer play_arrow
  • question_answer76) Which of the following is reduced by\[{{H}_{2}}{{O}_{2}}\]?

    A)
     \[C{{l}_{2}}\]

    B)
     \[{{[Fe{{(CN)}_{6}}]}^{4-}}\]

    C)
     \[N{{H}_{2}}OH\]

    D)
     \[SO_{3}^{2-}\]

    View Answer play_arrow
  • question_answer77) The maximum energy molecular orbit filled by electron in nitrogen molecule is/are

    A)
     \[\sigma 2{{p}_{z}}\]

    B)
     \[\pi 2{{p}_{x}}\approx \pi 2{{p}_{y}}\]

    C)
     \[{{\pi }^{*}}2{{p}_{x}}\approx {{\pi }^{*}}2{{p}_{y}}\]

    D)
     \[{{\sigma }^{*}}2{{p}_{z}}\]

    View Answer play_arrow
  • question_answer78) Which of the following is correct order for density?

    A)
     \[Cs>Rb>K>Na\]

    B)
     \[Cs>Rb>Na>K\]

    C)
     \[Rb>Cs>K>Na\]

    D)
     \[Rb>Cs>Na>K\]

    View Answer play_arrow
  • question_answer79) The correct order of dipole moment is

    A)
     \[B{{F}_{3}}<{{H}_{2}}S<{{H}_{2}}O\]  

    B)
     \[{{H}_{2}}S<B{{F}_{3}}<{{H}_{2}}O\]

    C)
     \[{{H}_{2}}O<{{H}_{2}}S<B{{F}_{3}}\]  

    D)
     \[{{H}_{2}}O<B{{F}_{3}}<{{H}_{2}}S\]

    View Answer play_arrow
  • question_answer80) Which of the following bond has minimum bond energy?

    A)
     \[C-H\]

    B)
     \[N-H\]

    C)
     \[O-H\]

    D)
     \[F-H\]

    View Answer play_arrow
  • question_answer81) The intersection point of the normals drawn at the end points of latusrectum of the parabola\[{{x}^{2}}=-2y\]is

    A)
     \[\left( -\frac{1}{2},-\frac{3}{2} \right)\]

    B)
     \[\left( \frac{1}{2},-\frac{3}{2} \right)\]

    C)
     \[(0,-1)\]

    D)
     \[\left( 0,-\frac{3}{2} \right)\]

    View Answer play_arrow
  • question_answer82) The equation whose roots are reciprocal of the roots of the equation\[a{{x}^{2}}+bx+c=0,\]is

    A)
     \[b{{x}^{2}}+cx+a=0\]   

    B)
     \[b{{x}^{2}}+ax+c=0\]

    C)
     \[c{{x}^{2}}+ax+b=0\]   

    D)
     \[c{{x}^{2}}+bx+a=0\]

    View Answer play_arrow
  • question_answer83) If in the expansion of\[{{\left( 3x-\frac{2}{{{x}^{2}}} \right)}^{15}},\]rth term is independent of\[x,\]then value of r is

    A)
     11     

    B)
     10    

    C)
     9     

    D)
     12

    View Answer play_arrow
  • question_answer84) If\[f(x)=\left| \begin{matrix}    1+a & 1-ax & 1+a{{x}^{2}}  \\    1+b & 1+bx & 1+b{{x}^{2}}  \\    1+c & 1+cx & 1+c{{x}^{2}}  \\ \end{matrix} \right|,\]where\[a,b,c\] are non-zero Constants, then value of\[f(10)\] is

    A)
     \[10(b-a)(c-a)\]

    B)
     \[100(b-a)(c-b)(a-c)\]

    C)
     \[100\,abc\]

    D)
     0

    View Answer play_arrow
  • question_answer85) If\[^{n}{{C}_{12}}{{=}^{n}}{{C}_{8}},\]then value of\[^{n}{{C}_{19}}\]is

    A)
     1                

    B)
     20

    C)
     210              

    D)
     1540

    View Answer play_arrow
  • question_answer86) If\[\omega \]is a complex cube root of unity and\[A=\left[ \begin{matrix}    \omega  & 0  \\    0 & \omega   \\ \end{matrix} \right],\]then\[{{A}^{50}}\]is

    A)
     \[{{\omega }^{2}}A\]

    B)
     \[\omega A\]

    C)
     \[A\]

    D)
     \[0\]

    View Answer play_arrow
  • question_answer87) If sum of\[n\]terms of an AP is\[2n+3{{n}^{2}},\]then rth term is

    A)
     \[2r+3{{r}^{2}}\]           

    B)
     \[3{{r}^{2}}-4r+1\]

    C)
     \[6r-1\]             

    D)
     \[4r+1\]

    View Answer play_arrow
  • question_answer88) If A and B are square matrices of order\[3\times 3,\] then which of the following is true?

    A)
     \[AB=0\Rightarrow A=O\]or\[B=O\]

    B)
     \[det(2AB)=8\text{ (}detA)\text{ (}detB)\]

    C)
     \[{{A}^{2}}-{{E}^{2}}=(A+B)(A-B)\]

    D)
     \[det(A+B)=det(A)+det(B)\]

    View Answer play_arrow
  • question_answer89) If geometric mean and harmonic mean between two different numbers are 12 and \[\frac{48}{5}\]respectively, then one number is

    A)
     4                

    B)
     6

    C)
     8                

    D)
     10

    View Answer play_arrow
  • question_answer90) Let a, b, c are in GP and 4a,5b,4c are in AP such that\[a+b+c=70,\]then value of\[b\]is

    A)
     5     

    B)
     10     

    C)
     15     

    D)
     20

    View Answer play_arrow
  • question_answer91) The value of integral\[\int_{0}^{4}{|x-1|}dx\] is

    A)
     4      

    B)
     5       

    C)
     7      

    D)
     9

    View Answer play_arrow
  • question_answer92) If\[\frac{x-1}{1+i}+\frac{y-1}{1-i}=i,\]where\[x\]and y are real numbers and\[i=\sqrt{-1},\]then

    A)
     \[x=0\]            

    B)
     \[x<0\]

    C)
     \[x>0\]            

    D)
     None of these

    View Answer play_arrow
  • question_answer93) If a pair of two fair dice is thrown, then the probability of getting the sum 5 on both dice is

    A)
     \[\frac{5}{36}\]

    B)
     \[\frac{1}{12}\]

    C)
     \[\frac{1}{18}\]

    D)
     \[\frac{1}{9}\]

    View Answer play_arrow
  • question_answer94) If a fair coin is tossed 20 times and let we get head n times, then probability that 21 is odd, is

    A)
     \[\frac{1}{2}\]

    B)
     \[\frac{1}{6}\]

    C)
     \[\frac{5}{8}\]

    D)
     \[\frac{7}{8}\]

    View Answer play_arrow
  • question_answer95) If\[{{x}_{r}}=\cos \left( \frac{\pi }{{{2}^{r}}} \right)+i\sin \left( \frac{\pi }{{{2}^{r}}} \right),\]then value of\[{{x}_{1}}\,{{x}_{2}}\,\,{{x}_{3}}....\]is

    A)
     \[i\]

    B)
     \[1\]

    C)
     \[-1\]

    D)
     \[-i\]

    View Answer play_arrow
  • question_answer96) If\[{{\left( \frac{1+\cos \phi +i\sin \phi }{1+\cos \phi -i\sin \phi } \right)}^{n}}=u+iv,\]where u and v are real numbers, then u is

    A)
     \[n\cos \phi \]

    B)
     \[\cos n\phi \]

    C)
     \[\cos \left( \frac{n\phi }{2} \right)\]

    D)
     \[\sin \left( \frac{n\phi }{2} \right)\]

    View Answer play_arrow
  • question_answer97) If square root of \[-7+24i\] is \[x+iy,\] then \[x\] is

    A)
     ±1              

    B)
     ± 2

    C)
     ±3               

    D)
     ± 4

    View Answer play_arrow
  • question_answer98) If \[{{\tanh }^{-1}}(x-iy)=\frac{1}{2}{{\tanh }^{-1}}\left( \frac{2x}{1+{{x}^{2}}+{{y}^{2}}} \right)\] \[+\frac{i}{2}{{\tan }^{-1}}\left( \frac{2y}{1-{{x}^{2}}-{{y}^{2}}} \right)x,y\in R,\] then \[{{\tanh }^{-1}}(iy)\] is

    A)
      \[2\text{ }tan{{h}^{-1}}(y)\]        

    B)
      \[-2\text{ }tan{{h}^{-1}}(y)\]

    C)
      \[\text{i }ta{{n}^{-1}}y\]          

    D)
      \[\text{-i }ta{{n}^{-1}}(y)\]

    View Answer play_arrow
  • question_answer99) If \[f:R\to R\] is defined as \[f(x)={{(1-x)}^{1/3}}\] then \[{{f}^{-1}}(x)\] is

    A)
      \[{{(1-x)}^{-1/3}}\]

    B)
      \[{{(1-x)}^{3}}\]

    C)
      \[1-{{x}^{3}}\]

    D)
      \[1-{{x}^{1/3}}\]

    View Answer play_arrow
  • question_answer100) If \[{{x}^{y}}={{e}^{x-y}},x>0,\] then value of \[\frac{dy}{dx}\] at (1,1) is

    A)
     0

    B)
      \[\frac{1}{2}\]

    C)
     1

    D)
     2

    View Answer play_arrow
  • question_answer101) The value of \[\underset{x\to 0}{\mathop{\lim }}\,\frac{(1-\cos 2x)}{{{x}^{2}}}\]is

    A)
     doesn't exist       

    B)
     infinite

    C)
     0                  

    D)
     2

    View Answer play_arrow
  • question_answer102) The value of differentiation of\[{{e}^{{{x}^{2}}}}\]w.r.t. to \[{{e}^{2x-1}}\]at\[x=1\]is

    A)
     e     

    B)
     0      

    C)
     \[{{e}^{-1}}\]     

    D)
     1

    View Answer play_arrow
  • question_answer103) If function\[f(x),x\in R\]is differentiable and \[f(1)=1,\]then value of\[\underset{x\to 0}{\mathop{\lim }}\,\frac{(1-\cos 2x)}{{{x}^{2}}}\]is

    A)
     0                 

    B)
     1

    C)
     \[f'(1)\]            

    D)
     \[\infty \]

    View Answer play_arrow
  • question_answer104) The value of integral \[\int_{0}^{2a}{\frac{f(x)}{f(x)+f(2a-x)}}dx,\]where\[f(x)\]is a continuous function, is

    A)
     0                

    B)
     1

    C)
     \[a\]                

    D)
     \[2a\]

    View Answer play_arrow
  • question_answer105) The area of the region bounded by the curves \[y=ex,\text{ y}=lo{{g}_{e}}x\]and lines\[x=1,\text{ }x=2\]is

    A)
     \[{{(e-1)}^{2}}\]

    B)
     \[{{e}^{2}}-e+1\]

    C)
     \[{{e}^{2}}-e+1-2lo{{g}_{e}}2\]

    D)
     \[{{e}^{2}}+e-2lo{{g}_{e}}2\]

    View Answer play_arrow
  • question_answer106) If\[f(x)=sin\text{ }x+cos\text{ }x+1\]and \[g(x)={{x}^{2}}+x,\text{ }x\in R,\]then value of\[fog(x)\]at \[x=0\]is

    A)
     0      

    B)
     1        

    C)
     2     

    D)
     3

    View Answer play_arrow
  • question_answer107) The maximum value of function \[f(x)=\sin x(1+\cos x),x\in R\]is

    A)
     \[\frac{{{3}^{3/2}}}{4}\]

    B)
     \[\frac{{{3}^{5/3}}}{4}\]

    C)
     \[\frac{3}{2}\]

    D)
     \[\frac{{{3}^{7/5}}}{4}\]

    View Answer play_arrow
  • question_answer108) The normal at point (1,1) to the curve\[{{y}^{2}}={{x}^{3}}\]is parallel to the line

    A)
     \[3x-y-2=0\]    

    B)
     \[2x+3y-7=0\]

    C)
     \[2x-3y+1=0\]   

    D)
     \[2y-3x+1=0\]

    View Answer play_arrow
  • question_answer109) Function\[f(x)=|x-1|+|x-2|,x\in R\]is

    A)
     differentiable everywhere in R

    B)
     except\[x=1\]and\[x=2\]differentiable everywhere in R

    C)
     not continuous at\[x=1\]and\[x=2\]

    D)
     increasing in\[R\]

    View Answer play_arrow
  • question_answer110) If\[f(1)=2\]and\[f'(1)=1,\]then the value of\[\underset{x\to 1}{\mathop{\lim }}\,\frac{2x-f(x)}{x-1}\]is

    A)
     \[-1\]            

    B)
     0

    C)
     \[1\]              

    D)
     2

    View Answer play_arrow
  • question_answer111) The distance of the point P (1,2,3) from the line which passes through the point A (4,2,2) and parallel to the vector\[2\hat{i}+3\hat{j}+6\hat{k},\]is

    A)
     \[\sqrt{10}\]     

    B)
     \[\sqrt{7}\]

    C)
     \[\sqrt{5}\]               

    D)
     \[1\]

    View Answer play_arrow
  • question_answer112) Let\[\overrightarrow{a}=2\hat{i}+\hat{k},\overrightarrow{b}=\hat{i}+\hat{i}+\hat{k}\]and\[\overrightarrow{c}=4\hat{i}-3\hat{j}+7\hat{k}\]. If r is a vector such that\[\overrightarrow{r}\times \overrightarrow{b}=\overrightarrow{c}\times \overrightarrow{b}\] and\[\overrightarrow{r}.\overrightarrow{a}=0,\]then value of\[\overrightarrow{r}.\overrightarrow{b}\]is

    A)
     7                

    B)
     \[-7\]

    C)
     \[-5\]               

    D)
     5

    View Answer play_arrow
  • question_answer113) A unit vector which is coplanar with \[\hat{i}+\hat{j}+2\hat{k}\]and\[\hat{i}+2\hat{j}+\hat{k}\]and perpendicular to \[\hat{i}+\hat{j}+\hat{k},\] is

    A)
     \[\frac{(-\hat{j}+\hat{k})}{\sqrt{2}}\]

    B)
     \[\frac{(\hat{k}-\hat{i})}{\sqrt{2}}\]

    C)
     \[\frac{(\hat{i}-\hat{j})}{\sqrt{2}}\]

    D)
     \[\frac{(\hat{i}-\hat{k})}{\sqrt{2}}\]

    View Answer play_arrow
  • question_answer114) If a line is inclined at\[45{}^\circ \]to both x-axis and\[y-\]axis, then the angle at which it is inclined to z-axis is

    A)
     \[45{}^\circ \]             

    B)
     \[60{}^\circ \]

    C)
     \[30{}^\circ \]              

    D)
     \[90{}^\circ \]

    View Answer play_arrow
  • question_answer115) The equation of plane which passes through the point\[(1,2,-1)\]and parallel to the plane \[x-y+2z=0\]is

    A)
     \[x-y+2z-3=0\]  

    B)
     \[x-y+2z+3=0\]

    C)
     \[x-y+2z+6=0\] 

    D)
     \[x-y+2z-7=0\]

    View Answer play_arrow
  • question_answer116) Let a plane passes through the point P(-1, -1,1) and also passes through a line joining the points Q (0,1,1) and R(Q,0,2). Then, the distance of plane from the point (0,0,0) is

    A)
     3                

    B)
     0

    C)
     \[1/\sqrt{6}\]             

    D)
     \[2/\sqrt{6}\]

    View Answer play_arrow
  • question_answer117) If\[f(x)\]and\[g(x),x\in R\]are continuous functions, then value of integral \[\int_{-\pi /2}^{\pi /2}{[\{f(x)+f(-x)\}\{g(x)-g(-x)\}]}\,dx\]is

    A)
     \[\pi \]                

    B)
     \[\pi /2\]

    C)
     \[1\]                

    D)
     0

    View Answer play_arrow
  • question_answer118) The equation of circle which touches the\[x-\]axis and 7-axis at points (1, 0) and (0, 1) respectively, is

    A)
     \[{{x}^{2}}+{{y}^{2}}-4y+3=0\]

    B)
     \[{{x}^{2}}+{{y}^{2}}-2y-2=0\]

    C)
     \[{{x}^{2}}+{{y}^{2}}-2x-27+2=0\]

    D)
     \[{{x}^{2}}+{{y}^{2}}-2x-2y+1=0\]

    View Answer play_arrow
  • question_answer119) If lines\[4x+3y=1,\text{ }y-x=5\]and\[kx+5y=1\]are concurrent, then value of k is

    A)
     0              

    B)
     1

    C)
     3               

    D)
     7

    View Answer play_arrow
  • question_answer120) The equation\[{{y}^{2}}-8y-x+19=0\]represents

    A)
     a parabola whose focus is\[\left( \frac{1}{4},0 \right)\]and directrix is\[x=-\frac{1}{4}\]

    B)
     a parabola whose vertex is (3, 4) and directrix is\[x=\frac{11}{4}\]

    C)
     a parabola whose focus is\[\left( \frac{13}{4},4 \right)\]and vertex is (0, 0)

    D)
     a curve which is not a parabola

    View Answer play_arrow

Study Package

   


You need to login to perform this action.
You will be redirected in 3 sec spinner